Pagina 1 di 1

Lavagna: 1 1/2 1/3 .... 1/100

Inviato: 14 gen 2010, 15:06
da Gogo Livorno
Su una lavagna sono scritti 100 numeri: 1, 1/2, 1/3.... fino a 1/100.
Si possono cancellare due numeri arbitrari A e B e riscrivere un solo numero, pari ad A+B+AB, al loro posto.
Dopo 99 operazioni di questo tipo resta sulla lavagna un numero solo.

Quale?

Inviato: 14 gen 2010, 20:12
da dario2994
Uhm definisco $ $a?b=a+b+ab $. L'operazione ? è commutativa e associativa. Per dimostrarlo basta farsi i conti xD
Per ipotesi il numero finale è una composizione di ? con i numeri in ordine sparso e con le parentesi pure. Ma per le proprietà dimostrate l'ordine non conta e le parentesi manco==> il numero finale equivale a (svolgere in ordine le operazioni):
$ $1?\frac{1}{1}?\frac{1}{2}?\frac{1}{3}?\dots ?\frac{1}{100} $
Ora svolgendo partendo da sinistra è facile convincersi che il risultato è 100 (viene banalmente per induzione ;) )

Come si può fare una dimostrazione del genere???
Bon l'idea di partenza è: il numero finale è unico (0)... questo viene in mente pensando che l'autore non è un pazzo e perciò il problema deve avere un senso.
Osservazione (1) Per (0) l'ordine in cui faccio le mosse non conta
Osservazione (2) Il risultato finale alla fin fine è considerabile una "grande composizione" di operazioni
Osservazione (3) Osservando (1),(2) ritorna in mente l'addizione (almeno a me) ma allora perchè non definire davvero l'operazione e dimostrarne le proprietà...
Una volta definita e dimostrate le proprietà si conclude facilmente... basta trovare un modo di calcolare il numero finale, che fortunatamente è facile ordinando nel modo più banale le operazioni ;)

Inviato: 14 gen 2010, 20:25
da OriginalBBB
Mi hai preceduto uffa. :D Mi è piaciuto questo esercizio

Inviato: 14 gen 2010, 20:54
da Gogo Livorno
dario2994 ha scritto:Uhm definisco $ $a?b=a+b+ab $. L'operazione ? è commutativa e associativa. Per dimostrarlo basta farsi i conti xD
Per ipotesi il numero finale è una composizione di ? con i numeri in ordine sparso e con le parentesi pure. Ma per le proprietà dimostrate l'ordine non conta e le parentesi manco==> il numero finale equivale a (svolgere in ordine le operazioni):
$ $1?\frac{1}{1}?\frac{1}{2}?\frac{1}{3}?\dots ?\frac{1}{100} $
Premesso che non ho a disposizione una soluzione "ufficiale" del problema, l'hai pensato esattamente come me, definendo l'operazione e dimostrandone le proprietà.

Cercavo però una maniera esclusivamente numerica (magari induttiva) per poter dimostrare il tutto.

Mi spiego, io cercavo di fare qualcosa tipo:

1) Come hai detto, se li metti in ordine decrescente si dimostra per induzione che il numero finale viene 100, o meglio, per qualsiasi serie da 1 a 1/n messa in ordine decrescente il numero finale viene n.

2) Adesso, esiste un modo per dimostrare che in realtà in qualsiasi ordine li prendo il risultato rimane n, con considerazioni esclusivamente numeriche?

Inviato: 14 gen 2010, 21:17
da dario2994
Uhm... non so se si possa fare, comunque pensandoci viene fuori che l'operazione così definita, se viene composta sugli elementi di A... fa uscire qualcosa tipo:
$ $\sum_{X\subseteq A}\prod_{i\in X}i $
Se si riesce a dimostrare questo poi si è concluso ;)

p.s. non ci provo perchè sono stanco, ma penso sia fattibile e dà anche una bella definizione dell'operazione ;)

Inviato: 14 gen 2010, 21:22
da Maioc92
dunque, se vuoi dimostrare che il risultato finale è univocamente determinato non credo tu possa saltare il dimostrare che valgono certe proprietà.

Invece c'è una cosa interessante da notare: è facile vedere che in generale dati n numeri $ a_1...a_n $ dopo aver applicato n-1 volte questo tipo di operazione si ottiene l'espressione $ \displaystyle\sum a_i+\sum_{i<j}a_ia_j+\sum_{i<j<k}a_ia_ja_k.....+(a_1a_2..a_n) $. Se n è pari ho un modo molto semplice di calcolare questo risultato: costruisco il polinomio $ P(x)=(x-a_1)(x-a_2)..(x-a_n) $, cioè il polinomio monico che ha per radici gli n numeri.
Per le relazioni coefficienti radici ho che l'espressione di partenza è uguale alla somma di tutti i coefficienti (tranne ovviamente l'1 del termine direttivo), dove però quelli di posto dispari vanno cambiati di segno. Poichè n è pari, per ottenere tale somma mi basta calcolare $ P(-1)-1 $ (dove il -1 è a causa del coefficiente direttivo, che va tolto dal risultato).
E' facile verificare che in questo caso si ha $ P(-1)-1=101-1=100 $

Inviato: 14 gen 2010, 22:04
da kn
Ma perché complicarsi la vita così?
Noto che $ \displaystyle~(a+1)(b+1)=(a+b+ab)+1 $
Da questo segue che il prodotto dei numeri che si ottengono aggiungendo 1 ai numeri sulla lavagna non cambia.
Segue che alla fine rimane solo un $ \displaystyle~n $ tale che $ \displaystyle~n+1=\prod_{i=1}^{100}(\frac{1}{i}+1)=\prod_{i=1}^{100}\frac{i+1}{i}=101 $ (è una produttoria telescopica). $ \displaystyle~n=100 $
La fonte?

Inviato: 14 gen 2010, 22:34
da Gogo Livorno
kn ha scritto: La fonte?
Libro delle Olimpiadi ;)

Inviato: 15 gen 2010, 10:31
da danielf
kn ha scritto:Ma perché complicarsi la vita così?
Noto che $ \displaystyle~(a+1)(b+1)=(a+b+ab)+1 $
Da questo segue che il prodotto dei numeri che si ottengono aggiungendo 1 ai numeri sulla lavagna non cambia.
Segue che alla fine rimane solo un $ \displaystyle~n $ tale che $ \displaystyle~n+1=\prod_{i=1}^{100}(\frac{1}{i}+1)=\prod_{i=1}^{100}\frac{i+1}{i}=101 $ (è una produttoria telescopica). $ \displaystyle~n=100 $
La fonte?
come si risolve tale produttoria?

Inviato: 16 gen 2010, 22:35
da kn
$ \displaystyle~\prod_{i=1}^{100}\frac{i+1}{i}=\frac{2}{1}\cdot\frac{3}{2}\cdot\frac{4}{3}\cdots\frac{100}{99}\cdot\frac{101}{100}=101 $, visto che il numeratore di ogni frazione si semplifica col denominatore della successiva.

Inviato: 17 gen 2010, 00:58
da Tibor Gallai
Non so se qualcuno l'ha notato, o l'ha scritto in modo più involuto...
Comunque, dati $ $a_1, a_2, \ldots, a_n $, il gioco termina con un

$ $\displaystyle\frac{\prod_i (a_i+1)}{\prod_i a_i}-1 $.

Inviato: 17 gen 2010, 12:10
da Gogo Livorno
Tibor Gallai ha scritto:Non so se qualcuno l'ha notato, o l'ha scritto in modo più involuto...
Comunque, dati $ $a_1, a_2, \ldots, a_n $, il gioco termina con un

$ $\displaystyle\frac{\prod_i (a_i+1)}{\prod_i a_i}-1 $.
non è la stessa soluzione che ha proposto kn?